25-01-2021 MCQs Test

10 Questions

For Latest Updates, Current Affairs & Knowledgeable Content.

1. Consider the following statements regarding Sundarbans:

  1. The delta is formed by the confluence of the Ganges, Brahmaputra and Meghna Rivers.
  2. It is a declared world heritage site by UNESCO as well as a Ramsar site.

Which of the statements given above is/are correct?

 
 
 
 

2. Recently, Smart Anti Airfield Weapon is developed by

 
 
 
 

3. Consider the following statements about Exercise Kavach:

  1. It is jointly conducted by Indian Army, Indian Navy, Indian Air Force and Indian Coast Guard.
  2. It is conducted under the aegis of Andaman and Nicobar Command.

Which of the statements given above is/are correct?

 
 
 
 

4. Recently, the ‘The Inequality Virus’ report is published by

 
 
 
 

5. Recently, a new species of ant namely, Ooceraea found in

 
 
 
 

6. Consider the following statements about the draft science policy:

  1. There will be a National STI Observatory as a central repository for all kinds of data related to and generated from the STI ecosystem.
  2. A dedicated portal namely Indian Science and Technology Archive of Research (INDSTA) will be created.

Which of the statements given above is/are correct?

 
 
 
 

7. Consider the following statements about Remote e-voting process:

  1. It can take place in person somewhere other than an assigned polling station.
  2. The votes may be sent by post or cast by an appointed proxy.

Which of the statements given above is/are incorrect?

 
 
 
 

8. Consider the following statements regarding Palk Strait:

  1. It connects the Bay of Bengal in the northeast with Palk Bay in the southwest.
  2. Its basin countries are India, Sri Lanka and Maldives.

Which of the statements given above is/are correct?

 
 
 
 

9. Recently, Situation Assessment Survey of Agricultural Households is released by

 
 
 
 

10. Consider the following statements about Joint Comprehensive Plan of Action (JCPOA):

  1. It is executed by the permanent members of UNSC.
  2. It aims to put some limits on nuclear weapons of North Korea.

Which of the statements given above is/are correct?

 
 
 
 
4 - 3 ?

Any Doubts ? Connect With Us.

Join Our Channels

For Latest Updates & Daily Current Affairs

Related Links

Connect With US Socially

Request Callback

Fill out the form, and we will be in touch shortly.

Call Now Button